Which one of the following statements could be true?

KGregory on August 25, 2020

Having Trouble setting this one up

So I am in my final stages....I take the LSAT on Saturday. I typically do very well on Logic games but I can not figure this one out. Can you help me with the setup?

Reply
Create a free account to read and take part in forum discussions.

Already have an account? log in

Victoria on August 26, 2020

Hi @KGregory,

Happy to help!

We know that cars are displayed on each floor of a three-floor building.

On each floor, the cars are either: (1) family cars or sports cars; (2) new cars or used cars; and (3) production models or research models.

Not F --> S
Not S --> F

Not N --> U
Not U --> N

Not P --> R
Not R --> P

We can set up our diagram as a grid with the floors on the left and the three car types along the bottom.

3: _ _ _
2: _ _ _
1: _ _ _
F/S N/U P/R

Now let's go through the conditions.

Rule 1: if the exhibition includes both F and S cars, then each F car is displayed on a lower-numbered floor than S cars.

Therefore, if there are both F and S cars at the exhibition, then S cars cannot be displayed on Floor 1 and F cars cannot be displayed on Floor 3.

Rule 2: there are no U R models

U --> Not R
R --> Not U

Rule 3: there are no R S cars

R --> Not S
S --> Not R

Rule 4: there are N cars on Floor 1

Rule 5: there are U cars on Floor 3

We know that there can be no U R models. Therefore, the cars on Floor 3 must be P models.

3: _ U P
2: _ _ _
1: _ N _
F/S N/U P/R

Now that we have our full diagram set up, let's go through the answer choices. We are looking for which answer choice could be true.

We can eliminate answer choices (A) and (C) because Rule 3 tells us that there are no R S cars.

We can eliminate answer choice (B) because Rule 2 tells us that there are no U R models.

We can also eliminate answer choice (E) because Rule 5 tells us that there are U cars on Floor 3, and we know that U cars must be P models. Therefore, at least Floor 3 has P models.

This means that answer choice (D) should be our answer, but let's double check.

3: S U P
2: F N R
1: F N P
F/S N/U P/R

Notice that this scenario meets all of our conditions:

1) Each F car is displayed on a lower-numbered floor than any S car.
2) The only R models at the exhibition are N cars.
3) The only S cars at the exhibition are P models.
4) There are N cars on Floor 1.
5) There are U cars on Floor 3.

There is at least one scenario (outlined above) where there are R models on exactly one floor. Therefore, answer choice (D) could be true and is our correct answer.

Hope this helps! Best of luck on the LSAT and please don't hesitate to let us know if you have any further questions before Saturday.